Weaken The Argument

This topic has expert replies
Master | Next Rank: 500 Posts
Posts: 135
Joined: Tue Oct 13, 2009 10:27 am
Thanked: 3 times

Weaken The Argument

by boazkhan » Fri Aug 06, 2010 4:13 pm
An ingredient in marijuana known as THC has been found to inactivate herpesviruses in experiments. In previous experiments researchers found that inactivated herpesviruses can convert healthy cells into cancer cells. It can be concluded that the use of marijuana can cause cancer.

Which one of the following, if true, most seriously
weakens the argument?

(A) Several teams of scientists performed the various
experiments and all of the teams had similar
results.
(B) The carcinogenic effect of THC could be
neutralized by the other ingredients found in
marijuana.
(C) When THC kills herpesviruses it weakens the
immune system, and it might thus diminish the
body's ability to fight other viruses, including
viruses linked to cancers.
(D) If chemists modify the structure of THC, THC
can be safely incorporated into medications to
prevent herpes.
(E) To lessen the undesirable side effects of
chemotherapy, the use of marijuana has been
recommended for cancer patients who are free of
the herpesvirus.

User avatar
Master | Next Rank: 500 Posts
Posts: 108
Joined: Fri Jul 09, 2010 8:15 am
Location: 127.0.0.1
Thanked: 15 times

by gmatrix » Fri Aug 06, 2010 5:59 pm
conclusion:use of marijuana can cause cancer
premise 1: THC(An ingredient in marijuana) inactivates herpesviruses.
premise 2.inactivated herpesviruses can convert healthy cells into cancer cells

ans:B

A.strengthens.
B.carcinogenic effect of THC could be
neutralized by the other ingredients found in
marijuana.
-thus it weakens the conclusion-It can be concluded that the use of marijuana can cause cancer
C. strengthens
D.THC can be safely incorporated into medications to
prevent herpes
-no correlation with the conclusion
E. strengthens
Life is all about ass; you're either covering it, laughing it off, kicking it, kissing it, busting it, trying to get a piece of it, or behaving like one.

Master | Next Rank: 500 Posts
Posts: 292
Joined: Fri Jul 17, 2009 8:39 am
Thanked: 6 times
Followed by:1 members

by pnk » Fri Aug 06, 2010 6:09 pm
IMO E

Master | Next Rank: 500 Posts
Posts: 364
Joined: Tue Apr 20, 2010 5:13 am
Thanked: 31 times
Followed by:3 members

by FightWithGMAT » Fri Aug 06, 2010 11:11 pm
boazkhan wrote:An ingredient in marijuana known as THC has been found to inactivate herpesviruses in experiments. In previous experiments researchers found that inactivated herpesviruses can convert healthy cells into cancer cells. It can be concluded that the use of marijuana can cause cancer.

Which one of the following, if true, most seriously
weakens the argument?

(A) Several teams of scientists performed the various
experiments and all of the teams had similar
results.
(B) The carcinogenic effect of THC could be
neutralized by the other ingredients found in
marijuana.
(C) When THC kills herpesviruses it weakens the
immune system, and it might thus diminish the
body's ability to fight other viruses, including
viruses linked to cancers.
(D) If chemists modify the structure of THC, THC
can be safely incorporated into medications to
prevent herpes.
(E) To lessen the undesirable side effects of
chemotherapy, the use of marijuana has been
recommended for cancer patients who are free of
the herpesvirus.
Yes It is E .

Conclusion says that use of marijuana causes cancer. E denies it, saying that it is not marijuana that causes cancer, but something else.

Master | Next Rank: 500 Posts
Posts: 141
Joined: Sat Feb 28, 2009 8:19 am
Thanked: 1 times

by getso » Fri Aug 06, 2010 11:34 pm
IMO [spoiler]<B>[/spoiler]

User avatar
Legendary Member
Posts: 1261
Joined: Sun Sep 14, 2008 3:46 am
Thanked: 27 times
GMAT Score:570

by reply2spg » Fri Aug 06, 2010 11:54 pm
Premise -
THC in marijuana inactivate herpesviruses.
In previous experiments researchers found that inactivated herpesviruses can convert healthy cells into cancer cells

Conclusion -
the use of marijuana can cause cancer

boazkhan wrote:An ingredient in marijuana known as THC has been found to inactivate herpesviruses in experiments. In previous experiments researchers found that inactivated herpesviruses can convert healthy cells into cancer cells. It can be concluded that the use of marijuana can cause cancer.

Which one of the following, if true, most seriously
weakens the argument?

(A) Several teams of scientists performed the various experiments and all of the teams had similar results. - Incorrect, this actually strengthen the conclusion. If several team did the experiment and concluded the same then marijuana causes the cancer.
(B) The carcinogenic effect of THC could be neutralized by the other ingredients found in marijuana. - Incorrect, Stick to the conclusion. This is trap. Conclusion does not say THC causes cancer, it says marijuana causes cancer. What to do with THC's neutralization?
(C) When THC kills herpesviruses it weakens the immune system, and it might thus diminish the body's ability to fight other viruses, including viruses linked to cancers. - Incorrect, this strengthens the argument
(D) If chemists modify the structure of THC, THC can be safely incorporated into medications to prevent herpes. - Incorrect, what to do with this?
(E) To lessen the undesirable side effects of chemotherapy, the use of marijuana has been recommended for cancer patients who are free of the herpesvirus. - Correct, marijuana is recommended for cancer patients to lessen the undesirable side effects of chemotherapy. If marijuana is given to the cancer patients then these people already have cancer before even exposing to marijuana. In that case how come marijuana causes the cancer? This is the weakening sentence
Sudhanshu
(have lot of things to learn from all of you)

User avatar
Legendary Member
Posts: 1261
Joined: Sun Sep 14, 2008 3:46 am
Thanked: 27 times
GMAT Score:570

by reply2spg » Sat Aug 07, 2010 12:25 am
boazkhan - Please cite source of this question while mentioning OA. If answer is B, then I would be shocked. Please and please cite the source.
Sudhanshu
(have lot of things to learn from all of you)

User avatar
Senior | Next Rank: 100 Posts
Posts: 44
Joined: Tue Jun 22, 2010 12:15 am
Thanked: 3 times

by showbiz » Sat Aug 07, 2010 1:13 am
https://gmatclub.com/forum/cr-question-m ... 43023.html

I hope its ok to post links from competitive forums.....

But this man says the answer is B. It was my guess. Time to get shocked :D

Master | Next Rank: 500 Posts
Posts: 203
Joined: Mon Mar 08, 2010 2:08 am
Thanked: 6 times

by SmarpanGamt » Sat Aug 07, 2010 3:04 am
showbiz wrote:https://gmatclub.com/forum/cr-question-m ... 43023.html

I hope its ok to post links from competitive forums.....

But this man says the answer is B. It was my guess. Time to get shocked :D

Hillariours Both you Showbiz and Sudhanshu....

It is a problem of casualty and THC of marijuana is casully linked for the cuase of cancer.

Therefore " B" seems to be the Bet.

By the way what is OA

User avatar
Legendary Member
Posts: 1261
Joined: Sun Sep 14, 2008 3:46 am
Thanked: 27 times
GMAT Score:570

by reply2spg » Sat Aug 07, 2010 7:30 am
I have already checked that site and therefore I asked to cite source of this question. Any website can never be the source.
showbiz wrote:https://gmatclub.com/forum/cr-question-m ... 43023.html

I hope its ok to post links from competitive forums.....

But this man says the answer is B. It was my guess. Time to get shocked :D
Sudhanshu
(have lot of things to learn from all of you)

User avatar
Senior | Next Rank: 100 Posts
Posts: 44
Joined: Tue Jun 22, 2010 12:15 am
Thanked: 3 times

by showbiz » Sat Aug 07, 2010 7:33 am
reply2spg wrote:I have already checked that site and therefore I asked to cite source of this question. Any website can never be the source.
showbiz wrote:https://gmatclub.com/forum/cr-question-m ... 43023.html

I hope its ok to post links from competitive forums.....

But this man says the answer is B. It was my guess. Time to get shocked :D
My apologies. I need to know the answer to this now.....

Legendary Member
Posts: 995
Joined: Tue Apr 13, 2010 11:56 pm
Thanked: 31 times
Followed by:1 members

by paes » Sat Aug 07, 2010 8:21 am
reply2spg wrote:boazkhan - Please cite source of this question while mentioning OA. If answer is B, then I would be shocked. Please and please cite the source.
The answer is straight B.
I read your analysis and seems that you are taking argument and E in wrong fashion
Read the argument again and I hope you will find the mistake what you are doing.

User avatar
Legendary Member
Posts: 1261
Joined: Sun Sep 14, 2008 3:46 am
Thanked: 27 times
GMAT Score:570

by reply2spg » Sat Aug 07, 2010 8:50 am
Paes - could you please help me understand what is wrong with E?
paes wrote:
reply2spg wrote:boazkhan - Please cite source of this question while mentioning OA. If answer is B, then I would be shocked. Please and please cite the source.
The answer is straight B.
I read your analysis and seems that you are taking argument and E in wrong fashion
Read the argument again and I hope you will find the mistake what you are doing.
Sudhanshu
(have lot of things to learn from all of you)

Legendary Member
Posts: 995
Joined: Tue Apr 13, 2010 11:56 pm
Thanked: 31 times
Followed by:1 members

by paes » Sat Aug 07, 2010 9:06 am
let me rephrse the argument as:

Wine contains alcohol and some X.
Alcohol creates -> effect Y -> heart problem.
So Wine can create heart problem.

Now D says :
X can deactivate the effect of alcohol.

hence:
Wine can create heart problem. -> wrong conclusion

Now come on E :

E says,
(To decrease some side effcet) wine is recommended to heart patients . : so what, it is just an information. It doesn't say that wine can't create the heart problem.

User avatar
Legendary Member
Posts: 1261
Joined: Sun Sep 14, 2008 3:46 am
Thanked: 27 times
GMAT Score:570

by reply2spg » Sat Aug 07, 2010 10:36 am
Frankly speaking I was struggling between B and E. However, before selecting E, I felt bit hard to eliminate B. Will explain you why?

Passage tell you that Marijuana has THC, you agree?
Option says that Marijuana contains one more ingredient, now tell me how can you assume that marijuana contains only 2 ingredients?

Remember passage tells you that THC has been found to inactivate herpesviruses, but how do we know that THC is the only thing which inactivate herpesviruses????

As per passage and option if Marijuana contains 3rd ingredient say XYZ and that is also capable of causing cancer. Now MArijuana contains THC, ABC (Which neutralized THC) and XYZ. In this case conclusion is still fine saying that marijuana (XYZ) causes cancer.

Marijuana is super set and we do not know what all subset it has. I am making this statement along with the passage and with option B. Conclusion says super set causes Cancer. We need to tell that no that is not correct.

E says that doctor is recommending marijuana to cancer patients, then how can it causes cancer? If aspirin is recommended by doctor for headache, then how can aspirin causes headache?????

I really appreciate you views on this question and I am puzzled by this question. However, we don't know source and we do not have any official answer for this question. In that case I can not conclude about the answer.

Hope you understand :)
paes wrote:let me rephrse the argument as:

Wine contains alcohol and some X.
Alcohol creates -> effect Y -> heart problem.
So Wine can create heart problem.

Now D says :
X can deactivate the effect of alcohol.

hence:
Wine can create heart problem. -> wrong conclusion

Now come on E :

E says,
(To decrease some side effcet) wine is recommended to heart patients . : so what, it is just an information. It doesn't say that wine can't create the heart problem.
Sudhanshu
(have lot of things to learn from all of you)